Προετοιμασία για JBMO (2)

Συντονιστές: cretanman, ΔΗΜΗΤΡΗΣ ΙΩΑΝΝΟΥ, socrates

Datis-Kalali
Δημοσιεύσεις: 117
Εγγραφή: Δευ Δεκ 12, 2016 5:33 pm
Τοποθεσία: Λευκωσία

Προετοιμασία για JBMO (2)

#1

Μη αναγνωσμένη δημοσίευση από Datis-Kalali » Τρί Μάιος 16, 2017 8:59 pm

Προβλήμα 1
Να βρείτε όλες τις ακέραιες λύσεις (x,y,z) της εξίσωσης
28^x=19^y+87^z

Προβλήμα 2
Αν a,b,c είναι θετικοί πραγματικοί αριθμοί , να βρείτε την μεγιστή τιμή του k, έτσι ώστε
\frac{a^3}{ka^2+b^2}+\frac{b^3}{kb^2+a^2}+\frac{c^3}{kc^2+a^2} \ge \frac{a+b+c}{1+k}
Προβλήμα 3
Ο εγγεγραμμένος κύκλος του σκαληνού τριγώνου ABC, με \angle{BAC}=60, εφάπτεται των πλευρών AB και AC στα σημεία E και F αντίστοιχα. Το σημείο M είναι το μέσο της ευθείας BC, και το σημείο I είναι το εγκέντρο του του τριγώνου ABC. Αν οι ευθείες BI και CI τέμνουν η ευθεία, EFστα σημεία P και Q αντίστοιχα, να δείξετε ότι το τρίγωνο PMQ είναι ισόπλευρο.



Λέξεις Κλειδιά:
Άβαταρ μέλους
Ορέστης Λιγνός
Δημοσιεύσεις: 1835
Εγγραφή: Κυρ Μάιος 08, 2016 7:19 pm
Τοποθεσία: Χαλάνδρι Αττικής
Επικοινωνία:

Re: Προετοιμασία για JBMO (2)

#2

Μη αναγνωσμένη δημοσίευση από Ορέστης Λιγνός » Τρί Μάιος 16, 2017 10:03 pm

Datis-Kalali έγραψε: Προβλήμα 3
Ο εγγεγραμμένος κύκλος του σκαληνού τριγώνου ABC, με \angle{BAC}=60, εφάπτεται των πλευρών AB και AC στα σημεία E και F αντίστοιχα. Το σημείο M είναι το μέσο της ευθείας BC, και το σημείο I είναι το εγκέντρο του του τριγώνου ABC. Αν οι ευθείες BI και CI τέμνουν η ευθεία, EFστα σημεία P και Q αντίστοιχα, να δείξετε ότι το τρίγωνο PMQ είναι ισόπλευρο.
Από εδώ είναι \widehat{BQC}=\widehat{BPC}=90^\circ.

Είναι BM=MC, άρα QM=MB=MC=PM (1).

Είναι \widehat{A}=60^\circ \Rightarrow \widehat{BIC}=120^\circ \mathop \Rightarrow \limits^{BQ \perp IQ} \widehat{QBP}=30^\circ (2).

Από MB=MQ=MP \Rightarrow M \, \textnormal{\gr περίκεντρο του} \, \vartriangle BEP \Rightarrow \widehat{QMP}=2\widehat{QBP} \mathop = \limits^{(2)} 60^\circ, άρα \widehat{QMP}=60^\circ (3).

Από (1), (3) QM=MP, \widehat{QMP}=60^\circ \Rightarrow \vartriangle QMP \, \textnormal{\gr ισόπλευρο}.


Κερδίζουμε ό,τι τολμούμε!
Άβαταρ μέλους
Ορέστης Λιγνός
Δημοσιεύσεις: 1835
Εγγραφή: Κυρ Μάιος 08, 2016 7:19 pm
Τοποθεσία: Χαλάνδρι Αττικής
Επικοινωνία:

Re: Προετοιμασία για JBMO (2)

#3

Μη αναγνωσμένη δημοσίευση από Ορέστης Λιγνός » Τρί Μάιος 16, 2017 11:50 pm

Datis-Kalali έγραψε: Προβλήμα 2
Αν a,b,c είναι θετικοί πραγματικοί αριθμοί , να βρείτε την μεγιστή τιμή του k, έτσι ώστε
\frac{a^3}{ka^2+b^2}+\frac{b^3}{kb^2+a^2}+\frac{c^3}{kc^2+a^2} \ge \frac{a+b+c}{1+k} (1)
Η λύση είναι λανθασμένη.


Θα δείξουμε ότι για κάθε k \geqslant 0 ισχύει η (1).

Είναι \displaystyle (1) \mathop \Rightarrow \limits^{ \cdot k} \sum \dfrac{ka^3}{ka^2+b^2} \geqslant \dfrac{k(a+b+c)}{k+1} (2)

Όμως, \dfrac{ka^3}{ka^2+b^2}=a-\dfrac{ab^2}{ka^2+b^2} και τα κυκλικά, άρα

\displaystyle(2) \Rightarrow \sum (a-\dfrac{ab^2}{ka^2+b^2}) \geqslant \dfrac{k(a+b+c)}{k+1} \Rightarrow \sum \dfrac{ab^2}{ka^2+b^2} \leqslant \sum a -\dfrac{k(a+b+c)}{k+1}=

\dfrac{a+b+c}{k+1}.

Από ΑΜ-ΓΜ, ka^2+b^2=a^2+a^2 + \ldots a^2+b^2+b^2 \geqslant (k+1) \sqrt[k+1]{a^{2k}b^2}, άρα \displaystyle \sum \dfrac{ab^2}{ka^2+b^2} \leqslant \sum \dfrac{\sqrt[k+1]{a^{1-k}b^{2k}}}{k+1} \leqslant \sum \dfrac{a(1-k)+2bk}{(k+1)^2}=

\dfrac{\displaystyle \sum [a(1-k)+2bk]}{(k+1)^2}=\dfrac{a+b+c}{k+1}, και η αρχική ανισότητα αποδείχτηκε.

Υ.Γ. Αν δεν κάνω λάθος η ανισότητα ισχύει για κάθε k \geqslant 0, οπότε πώς μπορεί να υπάρξει το \max k;

Η λύση είναι λανθασμένη

Ευχαριστώ τον Θανάση (socrates) για την επισήμανσή του στην επόμενη δημοσίευση.
τελευταία επεξεργασία από Ορέστης Λιγνός σε Τετ Μάιος 17, 2017 12:48 am, έχει επεξεργασθεί 1 φορά συνολικά.


Κερδίζουμε ό,τι τολμούμε!
socrates
Επιμελητής
Δημοσιεύσεις: 6461
Εγγραφή: Δευ Μαρ 09, 2009 1:47 pm
Τοποθεσία: Θεσσαλονίκη
Επικοινωνία:

Re: Προετοιμασία για JBMO (2)

#4

Μη αναγνωσμένη δημοσίευση από socrates » Τετ Μάιος 17, 2017 12:31 am

Ορέστης Λιγνός έγραψε:
Datis-Kalali έγραψε: Προβλήμα 2
Αν a,b,c είναι θετικοί πραγματικοί αριθμοί , να βρείτε την μεγιστή τιμή του k, έτσι ώστε
\frac{a^3}{ka^2+b^2}+\frac{b^3}{kb^2+a^2}+\frac{c^3}{kc^2+a^2} \ge \frac{a+b+c}{1+k} (1)

...

Από ΑΜ-ΓΜ, ka^2+b^2=a^2+a^2 + \ldots a^2+b^2+b^2 \geqslant (k+1) \sqrt[k+1]{a^{2k}b^2}, άρα \displaystyle \sum \dfrac{ab^2}{ka^2+b^2} \leqslant \sum \dfrac{\sqrt[k+1]{a^{1-k}b^{2k}}}{k+1} \leqslant \color{red} \sum \dfrac{a(1-k)+2bk}{(k+1)^2}=

\dfrac{\displaystyle \sum [a(1-k)+2bk]}{(k+1)^2}=\dfrac{a+b+c}{k+1}, και η αρχική ανισότητα αποδείχτηκε.

Υ.Γ. Αν δεν κάνω λάθος η ανισότητα ισχύει για κάθε k \geqslant 0, οπότε πώς μπορεί να υπάρξει το \max k;

Ορέστη, υπάρχει πρόβλημα στο επισημασμένο σημείο. Χρειαζόμαστε 1-k>0 σε αυτό το βήμα.


Θανάσης Κοντογεώργης
Άβαταρ μέλους
Διονύσιος Αδαμόπουλος
Δημοσιεύσεις: 807
Εγγραφή: Σάβ Μαρ 19, 2016 5:11 pm
Τοποθεσία: Πύργος Ηλείας

Re: Προετοιμασία για JBMO (2)

#5

Μη αναγνωσμένη δημοσίευση από Διονύσιος Αδαμόπουλος » Τετ Μάιος 17, 2017 12:40 am

Datis-Kalali έγραψε:Προβλήμα 1
Να βρείτε όλες τις ακέραιες λύσεις (x,y,z) της εξίσωσης
28^x=19^y+87^z
Έχουμε πως 29|87^z

Άρα πρέπει να ισχύει ότι:

28^x\equiv 19^y \pmod{29}

Όμως ισχύει ότι 28^x\equiv \pm 1 \pmod{29}

Άρα πρέπει 19^y\equiv \pm 1 \pmod{29}

Όμως το -1 δεν είναι υπόλοιπο του 19^y με το 29 (Αν δεν κάνω λάθος... :roll: )

Άρα αναγκαστικά έχουμε πως 28^x\equiv 19^y\equiv 1 \pmod{29}.

Έπεται λοιπόν πως x άρτιος. Έστω x=2k.

Τότε η εξίσωση γράφεται 28^{2k}=19^y+87^z\Leftrightarrow 784^k=19^y+87^z

Διακρίνουμε τις περιπτώσεις:

Αν z άρτιος, έστω z=2l:

Η εξίσωση γράφεται:

784^k=19^y+7569^l

Έχουμε πως 784^k\equiv \pm 1 \pmod{5}, 19^y\equiv \pm 1 \pmod{5}, 7569^l\equiv \pm 1 \pmod{5}

Πρέπει όμως 19^y+7569^l-784^k\equiv 0 \pmod{5}, που είναι άτοπο σε όλες τις παραπάνω περιπτώσεις.

Αν z περιττός:

Έχουμε πως 4|19^y+18^z και αφού z περιττός έχουμε πως 18^z\equiv -1 \pmod{4}. Άρα 19^y\equiv 1 \pmod{4}, δηλαδή y άρτιος, έστω y=2m.

Η εξίσωση γίνεται 28^{2k}-19^{2m}=87^z\Leftrightarrow (28^k-19^m)(28^k+19^m)=87^z

Όμως το 29 διαιρεί μόνο μια από τις δύο παρενθέσεις, όμοια και το 3. Άρα μια από τις δύο θα είναι 29^z και η άλλη 3^z. Προφανώς έχουμε πως 28^k+19^m=29^z και ότι 28^k-19^m=3^z, άρα 2\cdot 28^k=29^z+3^z.

Από το θεώρημα LTE έχουμε πως το 2 διαιρεί το 29^z+3^z σε μέγιστη δύναμη 5, καθώς ο z είναι περιττός. Όμως το 2 διαιρεί το 2\cdot 28^k σε μέγιστη δύναμη 2k+1. Άρα αναγκαστικά k=2.

Πρέπει λοιπόν να ισχύει ότι 1568=29^z+3^z, που δεν έχει λύσεις.

Επομένως η εξίσωση είναι άλυτη στους ακεραίους.


Houston, we have a problem!
Άβαταρ μέλους
Ανδρέας Πούλος
Δημοσιεύσεις: 1494
Εγγραφή: Κυρ Μαρ 01, 2009 10:47 pm
Τοποθεσία: ΘΕΣΣΑΛΟΝΙΚΗ
Επικοινωνία:

Re: Προετοιμασία για JBMO (2)

#6

Μη αναγνωσμένη δημοσίευση από Ανδρέας Πούλος » Τετ Μάιος 17, 2017 2:01 am

Απάντηση στο 1ο ΘΕΜΑ.

Θα εργαστούμε με τα ισοϋπόλοιπα του 9.
28\equiv 1(mod9) \Rightarrow 28^{x}\equiv 1(mod9)) (1)
19\equiv 1(mod 9) \Rightarrow 19^{y}\equiv 1(mod 9)) (2)
87\equiv -3(mod 9) \Rightarrow 87^{z}\equiv (-3)^{z})(mod 9)) (3)
Από (1), (2) και (3) προκύπτει ότι 1\equiv 1+ (-3)^{z})(mod 9)), το οποίο δεν ισχύει για κανέναν ακέραιο.
Άρα, η εξίσωση είναι αδύνατη στο σύνολο των ακεραίων.


Άβαταρ μέλους
Ορέστης Λιγνός
Δημοσιεύσεις: 1835
Εγγραφή: Κυρ Μάιος 08, 2016 7:19 pm
Τοποθεσία: Χαλάνδρι Αττικής
Επικοινωνία:

Re: Προετοιμασία για JBMO (2)

#7

Μη αναγνωσμένη δημοσίευση από Ορέστης Λιγνός » Τετ Μάιος 17, 2017 2:33 am

Ανδρέας Πούλος έγραψε:Απάντηση στο 1ο ΘΕΜΑ.


Από (1), (2) και (3) προκύπτει ότι 1\equiv 1+ (-3)^{z})(mod 9)), το οποίο δεν ισχύει για κανέναν ακέραιο.
Η παραπάνω ισοτιμία ισχύει για κάθε z άρτιο.


Κερδίζουμε ό,τι τολμούμε!
Άβαταρ μέλους
Doloros
Επιμελητής
Δημοσιεύσεις: 9850
Εγγραφή: Τρί Αύγ 07, 2012 4:09 am
Τοποθεσία: Ιεράπετρα Κρήτης

Re: Προετοιμασία για JBMO (2)

#8

Μη αναγνωσμένη δημοσίευση από Doloros » Τετ Μάιος 17, 2017 11:55 am

Datis-Kalali έγραψε:
Προβλήμα 3

Ο εγγεγραμμένος κύκλος του σκαληνού τριγώνου ABC, με \angle{BAC}=60, εφάπτεται των πλευρών AB και AC στα σημεία E και F αντίστοιχα. Το σημείο M είναι το μέσο της ευθείας BC, και το σημείο I είναι το εγκέντρο του του τριγώνου ABC. Αν οι ευθείες BI και CI τέμνουν η ευθεία, EFστα σημεία P και Q αντίστοιχα, να δείξετε ότι το τρίγωνο PMQ είναι ισόπλευρο.
προετοιμασία για JBMO.png
προετοιμασία για JBMO.png (50.21 KiB) Προβλήθηκε 1676 φορές
To \vartriangle AEF είναι προφανώς ισόπλευρο και άρα τα I\,\,\kappa \alpha \iota \,\,F βλέπουν υπό ίσες και

μάλιστα 60^\circ την PC , όθεν το τετράπλευρο IFPC είναι εγγράψιμο , συνεπώς

\boxed{\widehat {IPC} = 90^\circ } . Ομοίως \boxed{\widehat {IQB} = 90^\circ } . Τώρα θα είναι \boxed{MP = MQ = \frac{a}{2}} . Αφού όμως η

\widehat \theta είναι εξωτερική στο ισοσκελές τρίγωνο MBP θα είναι \widehat \theta  = \widehat B και ομοίως \widehat \omega  = \widehat C .

Μετά απ αυτά \widehat x = \widehat A = 60^\circ και άρα το \vartriangle MPQ είναι ισόπλευρο.


Άβαταρ μέλους
Ανδρέας Πούλος
Δημοσιεύσεις: 1494
Εγγραφή: Κυρ Μαρ 01, 2009 10:47 pm
Τοποθεσία: ΘΕΣΣΑΛΟΝΙΚΗ
Επικοινωνία:

Re: Προετοιμασία για JBMO (2)

#9

Μη αναγνωσμένη δημοσίευση από Ανδρέας Πούλος » Τετ Μάιος 17, 2017 11:59 am

Αγαπητέ Ορέστη,
Νόμιζα ότι το είχα σβήσει εχτές το βράδυ, είχα δει το λογικό κενό.
Αλλά, να που δεν έσβησα. Σήμερα, δεν νομίζω ότι έχει νόημα να το κάνω.
Επίσης, είχα κάνει και αποσύνδεση από το Φόρουμ, στο οποίο τώρα μπήκα κατευθείαν. Κάτι δεν έκανα σωστά.


Άβαταρ μέλους
Ανδρέας Πούλος
Δημοσιεύσεις: 1494
Εγγραφή: Κυρ Μαρ 01, 2009 10:47 pm
Τοποθεσία: ΘΕΣΣΑΛΟΝΙΚΗ
Επικοινωνία:

Re: Προετοιμασία για JBMO (2)

#10

Μη αναγνωσμένη δημοσίευση από Ανδρέας Πούλος » Τετ Μάιος 17, 2017 12:24 pm

Ορέστη,
βρήκα αυτό το πρόβλημα σε αυτή τη σελίδα
https://math.stackexchange.com/question ... n-integers


Άβαταρ μέλους
silouan
Επιμελητής
Δημοσιεύσεις: 1398
Εγγραφή: Τρί Ιαν 27, 2009 10:52 pm

Re: Προετοιμασία για JBMO (2)

#11

Μη αναγνωσμένη δημοσίευση από silouan » Τετ Μάιος 17, 2017 3:17 pm

Το πρόβλημα 3 είναι από τη ΒΜΟ του 2005.
https://artofproblemsolving.com/communi ... 934p224997

To πρόβλημα 1 είναι από την ΙΜΟ Longlist 1987 και μάλιστα προτάθηκε από την Ελλάδα.
https://artofproblemsolving.com/communi ... 83p2009076


Σιλουανός Μπραζιτίκος
knm2608
Δημοσιεύσεις: 26
Εγγραφή: Τετ Φεβ 15, 2017 7:00 pm

Re: Προετοιμασία για JBMO (2)

#12

Μη αναγνωσμένη δημοσίευση από knm2608 » Τετ Μάιος 17, 2017 3:50 pm

Το πρόβλημα 2 ήταν εδώ
https://artofproblemsolving.com/communi ... 513p815736
Η λύση βέβαια δεν είναι καθόλου κομψή.


Κωνσταντίνος Μεταξάς
Άβαταρ μέλους
Ορέστης Λιγνός
Δημοσιεύσεις: 1835
Εγγραφή: Κυρ Μάιος 08, 2016 7:19 pm
Τοποθεσία: Χαλάνδρι Αττικής
Επικοινωνία:

Re: Προετοιμασία για JBMO (2)

#13

Μη αναγνωσμένη δημοσίευση από Ορέστης Λιγνός » Τετ Μάιος 17, 2017 4:00 pm

knm2608 έγραψε:Το πρόβλημα 2 ήταν εδώ
https://artofproblemsolving.com/communi ... 513p815736
Η λύση βέβαια δεν είναι καθόλου κομψή.
:shock: Αναρωτιέμαι ποια είναι η λύση που έχει ο Datis - Kalali.


Κερδίζουμε ό,τι τολμούμε!
Άβαταρ μέλους
JimNt.
Δημοσιεύσεις: 590
Εγγραφή: Παρ Μάιος 20, 2016 3:00 pm

Re: Προετοιμασία για JBMO (2)

#14

Μη αναγνωσμένη δημοσίευση από JimNt. » Τετ Μάιος 17, 2017 4:22 pm

Ορέστης Λιγνός έγραψε:
knm2608 έγραψε:Το πρόβλημα 2 ήταν εδώ
https://artofproblemsolving.com/communi ... 513p815736
Η λύση βέβαια δεν είναι καθόλου κομψή.
:shock: Αναρωτιέμαι ποια είναι η λύση που έχει ο Datis - Kalali.
Γιατί είναι απαραίτητο να έχει λύση;


Bye :')
Άβαταρ μέλους
Demetres
Γενικός Συντονιστής
Δημοσιεύσεις: 8989
Εγγραφή: Δευ Ιαν 19, 2009 5:16 pm
Τοποθεσία: Λεμεσός/Πύλα
Επικοινωνία:

Re: Προετοιμασία για JBMO (2)

#15

Μη αναγνωσμένη δημοσίευση από Demetres » Τετ Μάιος 17, 2017 5:05 pm

JimNt. έγραψε:
Ορέστης Λιγνός έγραψε:
knm2608 έγραψε:Το πρόβλημα 2 ήταν εδώ
https://artofproblemsolving.com/communi ... 513p815736
Η λύση βέβαια δεν είναι καθόλου κομψή.
:shock: Αναρωτιέμαι ποια είναι η λύση που έχει ο Datis - Kalali.
Γιατί είναι απαραίτητο να έχει λύση;
Αν είναι σωστή η απάντηση στον σύνδεσμο, είναι ακατάλληλη και για Seniors. Πόσω μάλλον για Juniors...

Όταν δεν έχουμε λύση σε άσκηση που προτείνουμε πρέπει να το δηλώνουμε.


Άβαταρ μέλους
silouan
Επιμελητής
Δημοσιεύσεις: 1398
Εγγραφή: Τρί Ιαν 27, 2009 10:52 pm

Re: Προετοιμασία για JBMO (2)

#16

Μη αναγνωσμένη δημοσίευση από silouan » Τετ Μάιος 17, 2017 11:23 pm

Εγώ πιστεύω ότι βάσει της δυσκολίας και των πηγών των ασκήσεων, καμιά άσκηση δεν ανταποκρίνεται στο επίπεδο.


Σιλουανός Μπραζιτίκος
Άβαταρ μέλους
Ορέστης Λιγνός
Δημοσιεύσεις: 1835
Εγγραφή: Κυρ Μάιος 08, 2016 7:19 pm
Τοποθεσία: Χαλάνδρι Αττικής
Επικοινωνία:

Re: Προετοιμασία για JBMO (2)

#17

Μη αναγνωσμένη δημοσίευση από Ορέστης Λιγνός » Τετ Μάιος 17, 2017 11:35 pm

Συμφωνώ απόλυτα με τον Σιλουανό και να συμπληρώσω (αυτό δεν αφορά τον Datis - Kalali) ότι όποιοι προτείνουν ασκήσεις, να φροντίζουν να ανταποκρίνονται στον φάκελο που μπαίνουν, και να μην κάνουν ''επίδειξη'', βάζοντας ασκήσεις άσχετες, με ''τρελά'' θεωρήματα'', που απογοητεύουν τους μαθητές ( θέματα που δεν μπαίνουν ούτε σε IMO, ούτε είναι στο ύφος των διαγωνισμών!).

Αν θέλουμε να βάλουμε ''τρελά'' θέματα, θα πρέπει να φτιάξουμε έναν νέο φάκελο με τίτλο ''Πέρα από τα όρια''.

Και εκεί ο καθένας μπορεί να γράψει ό,τι ευφάνταστη ιδέα έχει ...


Κερδίζουμε ό,τι τολμούμε!
Απάντηση

Επιστροφή σε “Θέματα διαγωνισμών (ΕΜΕ, ΚΥΜΕ, BMO, JBMO, IMO, Kangaroo κλπ)”

Μέλη σε σύνδεση

Μέλη σε αυτήν τη Δ. Συζήτηση: Δεν υπάρχουν εγγεγραμμένα μέλη και 4 επισκέπτες